Difference between revisions of "2021 Fall AMC 10B Problems/Problem 5"
m (→Problem 5) |
m (→Solution 1) |
||
Line 5: | Line 5: | ||
==Solution 1== | ==Solution 1== | ||
− | We have <cmath>n=8^{2022}= \left(8^\frac{2}{3}\right)^{2022}=4^{3033}</cmath> | + | We have <cmath>n=8^{2022}= \left(8^\frac{2}{3}\right)^{2022}=4^{3033}.</cmath> |
− | Therefore, <cmath>\frac{n}4=\boxed{(\textbf{E})\:4^{3032}}</cmath> | + | Therefore, <cmath>\frac{n}4=\boxed{(\textbf{E})\:4^{3032}}.</cmath> |
~kingofpineapplz | ~kingofpineapplz |
Revision as of 20:54, 22 November 2021
Problem 5
Let . Which of the following is equal to
Solution 1
We have Therefore,
~kingofpineapplz